derivata temporale di una funzione

Messaggioda fonzimase » 11/02/2011, 14:00

salve a voi..devo calcolare come da oggetto la derivata temporale di una funzione, i cui parametri dipendono intrinsecamente dal tempo; premetto che non sono riuscito ad inserire il segno di moltiplicazione ( $ * $ ), che nella notazione che segue è stato sostituito dall'asterisco (*)
la funzione é:

$dot x$ * $sin^2$x$

dove $dot x$ ed "x" dipendono intrinsecamente dal tempo..e in cui $dot x$ è la derivata prima, temporale di x

il mio risultato (usando la regola di derivate composte) sarebbe:

$ddot x$ * $sin^2$x + $dot x$ * (2sinx * cosx)

ma al mio prof risulta:

$ddot x$ * $sin^2$x + $dot x$ * (2sinx * cosx * $dot x$)

noto subito che la differenza sta nel fatto che a lui compare un $dot x$ in più, quindi devo derivare anche la x rispetto al tempo;

quindi $sin^2$x è una derivata composta da derivare ancora rispetto al tempo?
fonzimase
New Member
New Member
 
Messaggio: 15 di 81
Iscritto il: 30/11/2009, 22:20

Messaggioda Akuma » 11/02/2011, 15:18

si come fa il prof è giusto. va derivato il seno, e poi la funzione interna che è $x$. è la regola di derivazione della funzione composta.
Più duro è il mondo, più ardente è l'onore
Avatar utente
Akuma
Junior Member
Junior Member
 
Messaggio: 199 di 491
Iscritto il: 08/06/2008, 10:34

Messaggioda Jackonzo » 01/03/2011, 20:41

Ciao a tutti, mi sono appena registrato e prima di porre la mia domanda volevo farvi i complimeti per il forum che mi ha già aiutato un paio di volte in passato. :D

Sono uno studente di ingegneria e sto preparando l'esame di meccanica razionale, ma ho un dubbio sulle derviate temporali.
Per spiegarmi meglio vi porto come esempio parte di un esercizio che ho svolto, dove con L indico la lagrangiana del sistema ed r e \( \displaystyle \wp \) [/tex] (dovrebbe essere "teta", ma non ho trovato il simbolo) sono i parametri lagrangiani.

\( \displaystyle L = 1/2 (\dot{r}^2+r^2\dot{\wp}^2)+\dot{r}\*e^r\wp^2+2\wp\dot{\wp}e^r-1/r^2 \)

Voglio calcolare l'euazione di Eulero - Lagrange (ad esempio rispetto a \( \displaystyle \wp \) ):

\( \displaystyle d/dt (\partial L/\partial \dot{\wp})=\partial L/\partial \wp \)

Il mio dubbio è su come si calcolano le derivate temporali, e siccome non riesco bene a spiegarmi continuo con l'esempio scrivendo le due soluzioni che ottengo in base a come derivo.

1)
\( \displaystyle \dot{r}^2\dot{\wp}+r^2\ddot{\wp}+ 2\dot{\wp}e^r+2\wp\dot{r}e^r = 2\dot{r}e^r\wp+2\dot{\wp}e^r \) che poi viene semplificata

2)
\( \displaystyle 2r\dot{r}\dot{\wp}+r^2\ddot{\wp}+2\dot{\wp}e^r+2\wp e^r\dot{r} = 2\dot{r}e^r\wp+2\dot{\wp}e^r \)

Quale delle due è giusta?

Grazie in anticipo :D

PS: Il colmo è se sono entrambe sbagliate :lol: :lol: :lol:
Jackonzo
Starting Member
Starting Member
 
Messaggio: 1 di 6
Iscritto il: 01/03/2011, 20:01

Messaggioda orazioster » 02/03/2011, 10:38

La seconda mi sembra giusta, dovrei controllare meglio.
Infatti $d/(dt)r^2=2r\dotr$.
orazioster
Senior Member
Senior Member
 
Messaggio: 467 di 1483
Iscritto il: 10/07/2008, 09:41

Messaggioda Jackonzo » 02/03/2011, 11:29

Grazie :)
Quindi la regola è: prima derivo "non rispetto al tempo", il risultato poi lo derivo rispetto al tempo?
Jackonzo
Starting Member
Starting Member
 
Messaggio: 2 di 6
Iscritto il: 01/03/2011, 20:01

Messaggioda ing@mate » 02/03/2011, 11:57

Non è proprio il risultato che devi derivare rispetto al tempo..
In questo caso, secondo la regola di derivazione di funzioni composte, il 'risultato' va moltiplicato per la funzione(dipendente dal tempo) derivata nel tempo.

In generale: $ [f(g(t))]'= f'(g(t)) * g'(t) $

Nel tuo caso $ f(g(t))=r^2 $ mentre $ g(t)= r $
Saro
ing@mate
New Member
New Member
 
Messaggio: 3 di 65
Iscritto il: 16/08/2010, 16:19

Messaggioda orazioster » 02/03/2011, 12:06

$"d"/("d"x) f "("$ $g "("$ $h "("..."("$ $n(x) ")"$ $")"$ $")"$ $")"$$= ("d"f)/("d"g)"*"("d"g)/("d"h)"*"...("d"n)/("d"x)$
orazioster
Senior Member
Senior Member
 
Messaggio: 470 di 1483
Iscritto il: 10/07/2008, 09:41

Messaggioda Jackonzo » 02/03/2011, 12:26

Capito.
Grazie a tutti! :)
Jackonzo
Starting Member
Starting Member
 
Messaggio: 3 di 6
Iscritto il: 01/03/2011, 20:01


Torna a Fisica, Fisica Matematica, Fisica applicata, Astronomia

Chi c’è in linea

Visitano il forum: Nessuno e 1 ospite